Đến nội dung

Hình ảnh

VMF's Marathon Đa thức Olympic

- - - - -

  • Please log in to reply
Chủ đề này có 31 trả lời

#1
Ego

Ego

    Thượng sĩ

  • Điều hành viên OLYMPIC
  • 296 Bài viết

Sau khi thông qua ý kiến các bạn, mình xin được nối tiếp các cuộc Marathon trước với Marathon Đa thức lần này. Mục tiêu là nhằm trau dồi, rèn luyện cho các bạn muốn tham gia các kỳ thi Olympic phổ thông.

Các chủ đề tiêu biểu mà các bạn có thể thảo luận:

  • Tính chia hết của đa thức
  • Quy tắc dấu Decartes
  • Các bài toán giải tích liên quan đến đa thức
  • Đa thức số học
  • Đa thức nguyên và các đa thức nhận giá trị nguyên
  • Tính bất khả quy/khả quy của đa thức
  • Phương trình hàm đa thức
  • Các bài toán xác định đa thức dựa trên các đặc trưng số học/các công thức nội suy
  • Đa thức lượng giác: Đa thức Chebyshev loại I, loại II
  • Các bài toán về đa thức nhiều biến
  • ...

Nội dung của cuộc thi này khá đơn giản, khi bạn giải đúng được bài toán hiện có thì bạn có thể đăng lên tại đây và mình sẽ cộng thêm cho các bạn một điểm, và các bạn có quyền được đề xuất bài toán mới. Như vậy ai giải thì người đó sẽ có quyền đề xuất, trừ khi bạn không biết đề xuất bài nào thì bạn có thể nhờ hỗ trợ.

 

Và một số quy định yêu cầu các bạn tuân thủ:

  1. Chỉ cho phép các bài toán trong phạm vi đa thức
  2. Ghi nguồn bài toán rõ ràng
  3. Không được phép giải bài toán của chính mình đề xuất, không được phép đề xuất các bài toán trong các cuộc thi chưa kết thúc (ví dụ như tạp chí toán học & tuổi trẻ,...)
  4. Không được spam, lời giải rõ ràng, cụ thể.
  5. Khi bạn giải bài toán thứ $n$ thì bạn đề xuất luôn bài toán thứ $n + 1$ (đánh đúng số thứ tự). Sau đây là mẫu:
    Lời giải bài $n$. ABCXYZ
    Bài toán $n + 1$. (Nguồn) Cho ba số $a, b, c$. Chứng minh rằng $3\mid abc$.
  6. Lưu ý không đăng các bài toán mở, các giả thuyết, ...
  7. Nếu một bài toán trong vòng $7$ ngày chưa ai giải được thì sẽ được đánh dấu lại và mình sẽ đăng bài toán tiếp theo. Bất cứ lúc nào bạn muốn đề xuất lời giải cho bài chưa được giải cũng được và sẽ được cộng hai điểm nếu như lời giải đúng. Ngoài ra nếu các bạn nghĩ mình có lời giải hay hơn của bạn trước tiên giải bài nào đó thì xin cứ đăng (sẽ chỉ cộng điểm cho bạn làm đúng và nhanh nhất), như vậy sẽ học hỏi lẫn nhau được nhiều hơn.
    Ngoài ra, trước khi hết hạn $4$ ngày của một bài toán chưa được giải thì mong các bạn không đề xuất bài toán mới.
  8. Yêu cầu các bài toán có độ khó nhất định, phải suy nghĩ mới làm được.
  9. Yêu cầu tuân thủ các quy định. Bài viết nào có tính chất spam sẽ bị xóa đi hoặc lời giải đúng nhưng không rõ ràng, lan man sẽ chỉ nhận được $0,5$ điểm.

Mình khuyến khích mọi người tự đưa lời giải của chính mình thay vì lời giải của người khác hoặc dẫn link lời giải.

Hi vọng các bạn tham gia và đón nhận  :D. Nếu các bài toán hay và lời giải đẹp thì ta sẽ tổng hợp thành một tài liệu nhỏ để tham khảo trong quá trình học Olympic, sẽ khá tốt.

Lưu ý: Các bạn khi đăng lời giải hãy để mọi người kiểm tra hộ bạn rồi hẳn đề xuất bài toán mới (kinh nghiệm của mình)
 


Bài viết đã được chỉnh sửa nội dung bởi Ego: 30-05-2016 - 17:53


#2
Ego

Ego

    Thượng sĩ

  • Điều hành viên OLYMPIC
  • 296 Bài viết

Bài toán hiện nay:

Bài toán 1 : Tìm tất cả các đa thức số hệ số nguyên thỏa mãn $a+b$ là số chính phương thì $P(a)+P(b)$ cũng là số chính phương, trong đó $a, b$ là các số tự nhiên (bangbang1412)
Bài toán 2. Cho $P(x) \in \mathbb{Z}[x]$ thỏa mãn $P(x)$ là số chính phương với mọi $x$ nguyên thì $P(x) = Q(x)^{2}$ với $Q(x) \in \mathbb{Z}[x]$.
Mở rộng: Liệu bạn có thể giải quyết cho trường hợp $P(x)$ là lũy thừa bậc $n$ của một số nguyên thì $P(x) = Q(x)^{n}$ với $Q(x)\in\mathbb{Z}[x]$ (Ego)


Bài viết đã được chỉnh sửa nội dung bởi Ego: 02-06-2016 - 23:00


#3
bangbang1412

bangbang1412

    Độc cô cầu bại

  • Phó Quản lý Toán Cao cấp
  • 1667 Bài viết

Bài toán 1 : Tìm tất cả các đa thức hệ số nguyên thỏa mãn $a+b$ là số chính phương thì $P(a)+P(b)$ cũng là số chính phương trong đó $a,b$ là các số tự nhiên .


$$[\Psi_f(\mathbb{1}_{X_{\eta}}) ] = \sum_{\varnothing \neq J} (-1)^{\left|J \right|-1} [\mathrm{M}_{X_{\sigma},c}^{\vee}(\widetilde{D}_J^{\circ} \times_k \mathbf{G}_{m,k}^{\left|J \right|-1})] \in K_0(\mathbf{SH}_{\mathfrak{M},ct}(X_{\sigma})).$$


#4
Ego

Ego

    Thượng sĩ

  • Điều hành viên OLYMPIC
  • 296 Bài viết

Bài toán 1 hiện vẫn chưa có lời giải nên mình được phép đề xuất bài toán tiếp theo:
Bài toán 2. Cho $P(x) \in \mathbb{Z}[x]$ thỏa mãn $P(x)$ là số chính phương với mọi $x$ nguyên thì $P(x) = Q(x)^{2}$ với $Q(x) \in \mathbb{Z}[x]$.
Mở rộng: Liệu bạn có thể giải quyết cho trường hợp $P(x)$ là lũy thừa bậc $n$ của một số nguyên thì $P(x) = Q(x)^{n}$ với $Q(x)\in\mathbb{Z}[x]$



#5
canhhoang30011999

canhhoang30011999

    Thiếu úy

  • Thành viên
  • 634 Bài viết

Bài toán 1 hiện vẫn chưa có lời giải nên mình được phép đề xuất bài toán tiếp theo:
Bài toán 2. Cho $P(x) \in \mathbb{Z}[x]$ thỏa mãn $P(x)$ là số chính phương với mọi $x$ nguyên thì $P(x) = Q(x)^{2}$ với $Q(x) \in \mathbb{Z}[x]$.
Mở rộng: Liệu bạn có thể giải quyết cho trường hợp $P(x)$ là lũy thừa bậc $n$ của một số nguyên thì $P(x) = Q(x)^{n}$ với $Q(x)\in\mathbb{Z}[x]$

bài này thì quen rồi

ta chứng minh bằng phản chứng

Giả sử $P(X)=Q(x)^{2}.p_{1}(x).p_{2}(x)...p_{k}(x)$ với $p_{i}(x)$ là các đa thức bất khả quy bậc lớn hơn 1

do $p_{i}(x)$ bất khả quy nên $p_{1}(x)$ và $p_{1}(x)'.p_{2}(x)...p_{k}(x)$ nguyên tố cùng nhau

nên theo định lý Bê-du thì tồn tại 2 đa thức $R(x),S(x)$ với hệ số nguyên sao cho

$R(x).p_{1}(x)+S(x).p_{1}(x)'.p_{2}(x)...p_{k}(x)=n$ với n là số nguyên 

chọn $p>n$ nguyên tố sao cho tồn tại a sao cho $p_{1}(a)$ chia hết cho p (luôn tồn tại p theo Schur) thì $p_{1}'(a)p_{2}(a)....p_{k}(a)$ không chia hết cho p

nếu $p_{1}(a)$ không chia hết cho $p^{2}$ ta suy ra vô lý vì khi đó $v_{p}(P(a))$ lẻ

nếu $p_{1}(a)$ chia hết cho $p^{2}$ thì xét $p_{1}(a+p) \equiv p_{1}(a)+p.p_{1}(a)'(mod p^{2})$ thì $p_{1}(a+p)$ không chia hết cho $p^{2}$

chứng minh tương tự trên thì ta suy ra mẫu thuẫn.

Vậy $P(x)=Q(x)^{2}$

Bài toán tổng quát thì cũng chứng minh tương tự 


Bài viết đã được chỉnh sửa nội dung bởi bangbang1412: 17-06-2016 - 17:14


#6
canhhoang30011999

canhhoang30011999

    Thiếu úy

  • Thành viên
  • 634 Bài viết

Bài 3 Chứng minh rằng đa thức $x^{n}-x-1$ bất khả quy trên $\mathbb{Z}[x]$ với mọi $n$ tự nhiên/


Bài viết đã được chỉnh sửa nội dung bởi Ego: 03-06-2016 - 22:37


#7
bangbang1412

bangbang1412

    Độc cô cầu bại

  • Phó Quản lý Toán Cao cấp
  • 1667 Bài viết

Bài 3 : Mình xài số phức không được nản quá tìm được cái này nhưng có vẻ lời giải của nó không rõ ràng ( lời giải cuối nhé , đoạn đồng nhất hệ số $x^{n}$) , mình sẽ cố gắng giải bằng cách khác nhưng cứ post cho m.n : 

http://math.stackexc...1-over-mathbb-q

À Hoàng làm lại bài $2$ được không cậu :( tớ hơi khó hiểu 


Bài viết đã được chỉnh sửa nội dung bởi bangbang1412: 03-06-2016 - 23:00

$$[\Psi_f(\mathbb{1}_{X_{\eta}}) ] = \sum_{\varnothing \neq J} (-1)^{\left|J \right|-1} [\mathrm{M}_{X_{\sigma},c}^{\vee}(\widetilde{D}_J^{\circ} \times_k \mathbf{G}_{m,k}^{\left|J \right|-1})] \in K_0(\mathbf{SH}_{\mathfrak{M},ct}(X_{\sigma})).$$


#8
baopbc

baopbc

    Himura Kenshin

  • Thành viên nổi bật 2016
  • 410 Bài viết

Topic trầm quá nhỉ? :( Nếu không ai đề nghị, mình xin đưa ra bài toán tiếp theo để topic được tiếp tục, nếu mọi người không đồng ý có thể xóa post này ra khỏi topic! :)

Bài toán 4. Tìm $P(x)\in \mathbb{Z}_{[x]}$ sao cho $P(x)$ nguyên tố với $\forall x\in \mathbb{N*}$.


Bài viết đã được chỉnh sửa nội dung bởi baopbc: 07-06-2016 - 12:26


#9
Ego

Ego

    Thượng sĩ

  • Điều hành viên OLYMPIC
  • 296 Bài viết

Bài toán 4. Tìm $P(x)\in \mathbb{Z}_{[x]}$ sao cho $P(x)$ nguyên tố với $\forall x\in \mathbb{N*}$.

Lời giải bài 4. Ta sẽ chứng minh $P(x) = p \quad\forall x \in \mathbb{R}$ là nghiệm duy nhất. Thật vậy, giả sử tồn tại $P$ khác hằng sao cho $\deg P \ge 1$. Ta có thể giả sử $P* > 0$ nhờ vào việc xét $-P(x)$ (cái này như nhau).
Vì vậy, với $x$ đủ lớn thì $P(x) > 1$. Để ý là $P(x + P(x)) - P(x) \vdots P(x) \implies P(x + P(x)) \vdots P(x)$. Điều này mâu thuẫn với giả thiết số nguyên tố.
Bài toán 5. (Irish MO 27th) Cho $n$ là số nguyên dương và $a_{1}, a_{2}, \cdots a_{n}$ là các số thực dương. Đặt $g(x) = (x + a_{1})(x + a_{2})\cdots (x + a_{n})$. Gọi $a_{0}$ là một số thực bất kỳ và đặt $f(x) = (x - a_{0}).g(x) = x^{n + 1} + b_{1}x^{n} + \cdots + b_{n + 1}$. Chứng minh rằng $b_{1}, b_{2}, \cdots b_{n + 1}$ đều âm nếu và chỉ nếu $a_{0} > a_{1} + a_{2} + \cdots a_{n}$


Bài viết đã được chỉnh sửa nội dung bởi Ego: 07-06-2016 - 22:12


#10
An Infinitesimal

An Infinitesimal

    Đại úy

  • Thành viên
  • 1803 Bài viết

Lời giải bài 4. Ta sẽ chứng minh $P(x) = p \quad\forall x \in \mathbb{R}$ là nghiệm duy nhất. Thật vậy, giả sử tồn tại $P$ khác hằng sao cho $\deg P \ge 1$. Ta có thể giả sử $P* > 0$ nhờ vào việc xét $-P(x)$ (cái này như nhau).
Vì vậy, với $x$ đủ lớn thì $P(x) > 1$. Để ý là $P(x + P(x)) - P(x) \vdots P(x) \implies P(x + P(x)) \vdots P(x)$. Điều này mâu thuẫn với giả thiết số nguyên tố.
Bài toán 5. (Irish MO 27th) Cho $n$ là số nguyên dương và $a_{1}, a_{2}, \cdots a_{n}$ là các số thực dương. Đặt $g(x) = (x + a_{1})(x + a_{2})\cdots (x + a_{n})$. Gọi $a_{0}$ là một số thực bất kỳ và đặt $f(x) = (x - a_{0}).g(x) = x^{n + 1} + b_{1}x^{n} + \cdots + b_{n + 1}$. Chứng minh rằng $b_{1}, b_{2}, \cdots b_{n + 1}$ đều âm nếu và chỉ nếu $a_{0} > a_{1} + a_{2} + \cdots a_{n}$

 

Lời giải bài toán 5.

Chiều  suy ra:

Theo định lý Viet, ta có

$-b_1= a_0-\sum_{k=1}^{n} a_k$.

Suy ra đpcm.

Chiều ngược lại:

 

Với $x=0$, ta có  $-a_0 \prod_{k=1}^{n}a_k= b_{n+1}$

Suy ra $b_{n+1}<0.$

Giả định $-a_1\le -a_2\le ...\le -a_n< a_0$.

Ta có $g(x)+(x-a_0)g'(x)= (n+1)x^n+ ...+b_{n-1}x+b_{n}.$

Dùng ĐL Lagrange, ta thấy tồn tại các số thực dương $y_i, i=\overline{0,n-1}$, sao cho $-a_1\le -y_1\le-a_2\le  -y_2...\le -a_n\le y_0\le  a_0$, và $f'(y_i)=0 \forall i=\overline{0,n-1}.$

 

Do đó \[\frac{1}{n+1}f'(x)= (x-y_0)\prod_{i=1}^{n}(x+y_i).\]

Đồng nhất hệ số $x^{n-1}$, ta sẽ thu được

\[y_{0}-\sum_{i=1}^{n-1} y_i= \frac{n}{n+1} \left(a_0- \sum_{i=1}^{n}a_i\right)>0.\]

 

 
Từ ý tưởng trên, ta sẽ dùng qui nạp để chỉ ra các hệ số $b_i$ đều âm.

Bài toán 6: (chưa có)


Bài viết đã được chỉnh sửa nội dung bởi vanchanh123: 09-06-2016 - 23:54

Đời người là một hành trình...


#11
bangbang1412

bangbang1412

    Độc cô cầu bại

  • Phó Quản lý Toán Cao cấp
  • 1667 Bài viết

Để tránh đóng băng topic mình xin phép đề xuất bài tiếp theo

Bài 6 :  ( Nguồn : VMF ) Gọi $F_n$ là số hạng thứ $n$ của dãy Fibonacci. Xét đa thức

$$P_n(x)=F_n^2x^n+F_{n+1}^2x^{n-1}+F_{n-2}^2x^{n-2}+...+F_1^2x+F_{n-1}^2$$

Chứng minh rằng với mọi số tự nhiên $n$ thì đa thức $P(x)$ luôn bất khả quy trên $\mathbb{Z}\left [ x \right ]$.


Bài viết đã được chỉnh sửa nội dung bởi bangbang1412: 11-06-2016 - 13:49

$$[\Psi_f(\mathbb{1}_{X_{\eta}}) ] = \sum_{\varnothing \neq J} (-1)^{\left|J \right|-1} [\mathrm{M}_{X_{\sigma},c}^{\vee}(\widetilde{D}_J^{\circ} \times_k \mathbf{G}_{m,k}^{\left|J \right|-1})] \in K_0(\mathbf{SH}_{\mathfrak{M},ct}(X_{\sigma})).$$


#12
bangbang1412

bangbang1412

    Độc cô cầu bại

  • Phó Quản lý Toán Cao cấp
  • 1667 Bài viết

:D Giải bài 3 :  Lời giải tìm trong một tài liệu của bạn hoanglong2k . Chứng minh $x^{n}-x-1$ bất khả quy trên $Z[x]$ với mọi $n\geq 2$ 

Bổ đề : Với mọi nghiệm phức $z$ của đa thức $P(x)=x^{n}-x-1$ chúng ta có bất đẳng thức sau:

$$2Re(z-\frac{1}{z})>\frac{1}{|z|^{2}}-1$$

Chứng minh : Ta đặt $z=r.e^{it}$ bất đẳng thức tương đương $(1+2rcost)(r^{2}-1)>0$ và do nó là nghiệm của $P(x)$ nên $$r^{2n}=|z|^{2n}=|z+1|^{2}=1+2rcost+r^{2}=>1+2rcost=r^{2n}-r^{2}$$

Bất đẳng thức trở thành 

$$(r^{2n}-r^{2})(r^{2}-1)>0$$ ( đúng )

Giả sử $P(x)=f(x)g(x)$ trong đó $degf,degg \geq 1$ và $f,g \in Z[x]$ , gọi các nghiệm phức của $P(x)$ là $z_{1},z_{2},....z_{n}$ sử dụng bổ đề ta có :

$$2\sum_{i=1}^{k}(z_{i}-\frac{1}{z_{i}}) > \sum_{i=1}^{k}\frac{1}{|z_{i}|^{2}}-k$$

Trong đó ta đã giả sử

$$f(x)=\prod_{i=1}^{k}(x-x_{i})$$

Mà ta lại theo $Viet$ có 

$$\prod_{i=1}^{k}|z_{i}|=|f(0)|=1$$ 

Theo bất đẳng thức $AM-GM$ thì:

$$\sum_{i=1}^{k}\frac{1}{|z_{i}|^{2}}-k\geq 0$$

Từ đó kéo theo

$$\sum_{i=1}^{k}Re(z_{i}-\frac{1}{z_{i}})>0$$

Mặt khác $f$ monic và hệ số nguyên nên 

$$\sum_{i=1}^{k}Re(z_{i}-\frac{1}{z_{i}})\geq 1$$

Lý luận tương tự với $g$ cộng lại thu được

$$Re(\sum_{i=1}^{n}(z_{i}-\frac{1}{z_{i}})\geq 2$$

Nhưng lại theo định lý $Vi-et$ thì

$$\sum_{i=1}^{n}(z_{i}-\frac{1}{z_{i}})=1$$

Từ đây suy ra mâu thuẫn , vậy ta có $P$ bất khả quy trên $Z[x]$


Bài viết đã được chỉnh sửa nội dung bởi bangbang1412: 11-06-2016 - 14:09

$$[\Psi_f(\mathbb{1}_{X_{\eta}}) ] = \sum_{\varnothing \neq J} (-1)^{\left|J \right|-1} [\mathrm{M}_{X_{\sigma},c}^{\vee}(\widetilde{D}_J^{\circ} \times_k \mathbf{G}_{m,k}^{\left|J \right|-1})] \in K_0(\mathbf{SH}_{\mathfrak{M},ct}(X_{\sigma})).$$


#13
bangbang1412

bangbang1412

    Độc cô cầu bại

  • Phó Quản lý Toán Cao cấp
  • 1667 Bài viết

Giải bài 1 : ( Nguyễn Tiến Khải , Phạm Khoa Bằng - THPT chuyên KHTN ) 

Bổ đề : Nếu $P(x) \in Z[x]$ là số chính phương với mọi $x$ thì nó là bình phương của một da thức khác ( đã là bài toán $2$ ) 

Quay trở lại bài toán nhận xét rằng $a+b$ chính phương thì $(a+b)x^{2}$ chính phương ,cố định $a,b$ xét đa thức $Q(x)=P(ax^{2})+P(bx^{2})$ là số chính phương với vô số $x$ nên là bình phương của đa thức $G(x)$ nào đó , đặt $G(x)=g_{m}x^{m}+...$ và $P(x)=p_{n}x^{n}+...$  đồng nhất hệ số bậc cao nhất ta suy ra $p_{n}(a^{n}+b^{n})=g^{2}$ , nghĩa là với mọi $a+b$ chính phương thì ta có điều trên . Cặp $(1,0)$ có $1+0$ chính phương suy ra với mọi $a+b$ scp thì $a^{n}+b^{n}$ là scp với $n \geq 1$ còn $n=0$ cho ta $P(x)=2k^{2}$. Với $(2,2)$ ta suy ra $2.2^{n}=2^{n+1}$ suy ra $n$ lẻ . Chọn $(a,b)=(3,1)$ ta có $3^{n}+1=u^{2}$ hay $3^{n}=(u-1)(u+1)$ , ta có $(3,2)=1$ và $gcd(u+1,u-1)|2$ , mà $3$ nguyên tố nên $u-1=1$ hay $u=2$ do đó $n=1$ . Đến đây dễ dàng giải quyết nốt , đặt $P(x)=kx$ thế thì cần có $k$ chính phương . Kết luận $P=k^{2}x,2k^{2}$


Bài viết đã được chỉnh sửa nội dung bởi bangbang1412: 24-07-2016 - 15:52

$$[\Psi_f(\mathbb{1}_{X_{\eta}}) ] = \sum_{\varnothing \neq J} (-1)^{\left|J \right|-1} [\mathrm{M}_{X_{\sigma},c}^{\vee}(\widetilde{D}_J^{\circ} \times_k \mathbf{G}_{m,k}^{\left|J \right|-1})] \in K_0(\mathbf{SH}_{\mathfrak{M},ct}(X_{\sigma})).$$


#14
anhquannbk

anhquannbk

    Sĩ quan

  • Thành viên
  • 477 Bài viết

Lời giải bài toán 6.

Ta có bổ đề sau: Cho $F_n$ là dãy $Fibonacci$. Khi đó với mọi $n$ nguyên dương ta có:

$F_{1}^2+F_{2}^2+......+F_{n}^2=F_n.F_{n+1}$

Chứng minh bổ đề:

Ta chứng minh quy nạp theo $n$, dễ thấy đẳng thức đúng với $n=1$. Giả sử đẳng thức đã cho đúng tới $n=k, k\in N$, tức là:

$F_{1}^2+F_{2}^2+.......+F_{k}^2=F_k.F_{k+1}.$

Xét $n=k+1$ ta có:

$F_{1}^2+F_{2}^2+......+F_{k+1}^2$

$=(F_{1}^2+F_{2}^2+.........+F_{k}^2) + F_{k+1}^2$

$= F_k.F_{k+1} +F_{k+1}^2$

$=F_{k+1}(F_k+F_{k+1})$

$=F_{k+1}.F_{k+2}$.

Vậy đẳng thức đúng với $n=k+1$

Bổ đề được chứng minh.

Ta có một tiêu chuẩn bất khả quy của đa thức như sau:

Tiêu chuẩn Perron:

Cho đa thức nguyên $P(x)= \sum_{i=0}^{k} a_{i}x^{i} $ có $a_0 \ne 0$. Khi đó nếu

$|a_{n-1}| > |a_0|+|a_1|+....+|a_{n-2}| +|a_n|$

thì đa thức này bất khả quy.

Tiêu chuẩn này được chứng minh trong nhiều sách và tài liệu.

Quay trở lại bài toán

Theo tiêu chuẩn $Perron$ ta chỉ cần chứng minh

$F_{n+1}^2 > F_{1}^2+F_{2}^2+.......+F_{n}^2$

Mà theo bổ đề trên thì ta quy về $F_{n+1}^2 > F_n.F_{n+1}$ hay $F_{n+1} >F_{n}$ (luôn đúng với mọi $n$ nguyên dương)

Vậy bài toán được chứng minh.


Bài viết đã được chỉnh sửa nội dung bởi anhquannbk: 12-06-2016 - 07:12


#15
anhquannbk

anhquannbk

    Sĩ quan

  • Thành viên
  • 477 Bài viết

Bài toán 7: (Sưu tầm) Cho $f(x)$ là đa thức nguyên có các nghiệm $ \alpha_1, \alpha_2,......, \alpha_n$.

Đặt $M=max|\alpha_i|$ với $ i \in \overline{1, n}.$

Chứng minh rằng nếu $f(x_0)$ là số nguyên tố với $x_0$ là số nguyên thỏa mãn $|x_0| > M+1$ thì $f(x)$ bất khả quy trên $\mathbb{Z}\left [ x \right ]$.


Bài viết đã được chỉnh sửa nội dung bởi anhquannbk: 12-06-2016 - 08:50


#16
bangbang1412

bangbang1412

    Độc cô cầu bại

  • Phó Quản lý Toán Cao cấp
  • 1667 Bài viết

Bài toán 7: (Sưu tầm) Cho $f(x)$ là đa thức nguyên có các nghiệm $ \alpha_1, \alpha_2,......, \alpha_n$.

Đặt $M=max|\alpha_i|$ với $ i \in \overline{1, n}.$

Chứng minh rằng nếu $f(x_0)$ là số nguyên tố với $x_0$ là số nguyên thỏa mãn $|x_0| > M+1$ thì $f(x)$ bất khả quy trên $\mathbb{Z}\left [ x \right ]$.

$M$ là max các hệ số hay nghiệm thế bạn .


$$[\Psi_f(\mathbb{1}_{X_{\eta}}) ] = \sum_{\varnothing \neq J} (-1)^{\left|J \right|-1} [\mathrm{M}_{X_{\sigma},c}^{\vee}(\widetilde{D}_J^{\circ} \times_k \mathbf{G}_{m,k}^{\left|J \right|-1})] \in K_0(\mathbf{SH}_{\mathfrak{M},ct}(X_{\sigma})).$$


#17
anhquannbk

anhquannbk

    Sĩ quan

  • Thành viên
  • 477 Bài viết

$M$ là max các hệ số hay nghiệm thế bạn .

$M$ là max của các nghiệm, mình đã sửa lại.



#18
canhhoang30011999

canhhoang30011999

    Thiếu úy

  • Thành viên
  • 634 Bài viết

Lời giải bài 7

Giả sử $f(x)$ khả quy thì tồn tại $g(x)$,$h(x)$$\in \mathbb{Z}[x]$ sao cho $f(x)=g(x).h(x)$($deg g,deg h>0) 

Khi đó $g(x_0)h(x_0)=p$ nên $|g(x_0)|=1$ hoặc $|h(x_0)|=1$ giả sử $|g(x_0)|=1$

Khi đó giả sử $\alpha _1,\alpha _2,...\alpha_k$ là nghiệm của $g(x)$ thì $|g(x_0)|=|a||(x-\alpha_1)||(x-\alpha_2)|...|(x-\alpha_k)|\geq (|x_0|-|\alpha_1|)...(|x|-|\alpha_k|)> 1$ (vô lý)

Vậy $f(x)$ bất khả quy


Bài viết đã được chỉnh sửa nội dung bởi canhhoang30011999: 12-06-2016 - 09:15


#19
canhhoang30011999

canhhoang30011999

    Thiếu úy

  • Thành viên
  • 634 Bài viết

Bài 8 

Chứng minh đa thức sau bkq

$X_p^{k}=\frac{x^{p^{k}}-1}{x^{p^{k-1}}-1}$ với $p$ nguyên tố


Bài viết đã được chỉnh sửa nội dung bởi bangbang1412: 12-06-2016 - 09:29


#20
bangbang1412

bangbang1412

    Độc cô cầu bại

  • Phó Quản lý Toán Cao cấp
  • 1667 Bài viết

Lời giải bài 8 :  

Đặt $x^{p^{k-1}} \to x $ xét đa thức $f(x) = \frac{x^{p}-1}{x-1}$ , đa thức này thì quen thuộc rồi , quy về $f(x)=\frac{(x+1)^{p}-1}{x}$ hay $f(x)=(x+1)^{p-1}+...+1$ bất khả quy . Ta có $f(x)=x^{p-1}+\binom{p}{1}x^{p-2}+....+p$ . Quy ngược lại $x^{p^{k-1}}$ cho ta $f(x)=x^{p^{k-1}(p-1)}+\binom{p}{1}x^{p^{k-1}(p-2)}+....+p$ . Áp dụng Eisenstein có $a_{p^{k-1}(p-1)}=1$ các hệ số còn lại là bội của $p$ ( bằng $0$ hoặc là bội của $p$ vì $p|\binom{p}{i}$ ) , mặt khác $p$ không chia hết $p^{2}$ nên có đpcm . 


Bài viết đã được chỉnh sửa nội dung bởi bangbang1412: 12-06-2016 - 09:42

$$[\Psi_f(\mathbb{1}_{X_{\eta}}) ] = \sum_{\varnothing \neq J} (-1)^{\left|J \right|-1} [\mathrm{M}_{X_{\sigma},c}^{\vee}(\widetilde{D}_J^{\circ} \times_k \mathbf{G}_{m,k}^{\left|J \right|-1})] \in K_0(\mathbf{SH}_{\mathfrak{M},ct}(X_{\sigma})).$$





0 người đang xem chủ đề

0 thành viên, 0 khách, 0 thành viên ẩn danh